Prove that <mstyle displaystyle="true" scriptlevel="0"> <munder> &#x2211;<!-- ∑ -->

Carly Cannon

Carly Cannon

Answered question

2022-07-04

Prove that 1 k < j n tan 2 ( k π 2 n + 1 ) tan 2 ( j π 2 n + 1 ) = ( 2 n + 1 4 )

Answer & Explanation

escampetaq5

escampetaq5

Beginner2022-07-05Added 12 answers

It is enough to recall Cauchy's proof of the Basel problem, relying on the identity:
(1) ( 2 n + 1 1 ) t n ( 2 n + 1 3 ) t n 1 + + ( 1 ) n ( 2 n + 1 2 n + 1 ) = k = 1 n ( t cot 2 k π 2 n + 1 )
If we consider the "reciprocal polynomial"
(2) ( 2 n + 1 2 n + 1 ) t n ( 2 n + 1 2 n 1 ) t n 1 + + ( 1 ) n ( 2 n + 1 1 ) = k = 1 n ( t tan 2 k π 2 n + 1 )
the original sum is just the second elementary symmetric polynomial of the roots, e 2 , that by Vieta's theorem is given by the coefficient of t n 2 in the LHS of (2), so:
(3) 1 j < k n tan 2 ( j π 2 n + 1 ) tan 2 ( k π 2 n + 1 ) = ( 2 n + 1 2 n 3 ) = ( 2 n + 1 4 )
as wanted.

Do you have a similar question?

Recalculate according to your conditions!

Ask your question.
Get an expert answer.

Let our experts help you. Answer in as fast as 15 minutes.

Didn't find what you were looking for?